2017年7月2日 星期日

106年桃連區內壢高中特招數學詳解



解:
$$\left( 1+\frac { 5 }{ 3 }  \right) \times \left( -3\frac { 3 }{ 4 }  \right) -\frac { 11 }{ 6 } \div \frac { 1 }{ 12 } =\frac { 8 }{ 3 } \times \left( \frac { -15 }{ 4 }  \right) -\frac { 11 }{ 6 } \times 12\\ =-10-22=-32$$,故選\(\bbox[red,2pt]{(D)}\)。



解:
$$2a+10b-c=2\times 3.7\times { 10 }^{ -4 }+10\times 5.1\times { 10 }^{ -5 }-4\times { 10 }^{ -4 }\\ =7.4\times { 10 }^{ -4 }+5.1\times { 10 }^{ -4 }-4\times { 10 }^{ -4 }=\left( 7.4+5.1-4 \right) \times { 10 }^{ -4 }\\ =8.5\times { 10 }^{ -4 }$$,故選\(\bbox[red,2pt]{(B)}\)。


解:
\(25-4a=0\Rightarrow a=\frac{25}{4}\),故選\(\bbox[red,2pt]{(D)}\)。


解:
$$原式={ \left( 3x^{ 2 }+5x-12 \right)  }^{ 2 }={ \left[ \left( 3x-4 \right) \left( x+3 \right)  \right]  }^{ 2 }$$,故選\(\bbox[red,2pt]{(B)}\)。


解:
分母越小則數值越大,且小於1的數開根號後變大。由於\({\sqrt{0.5}}^2>{0.6}^2\),所以\(\sqrt{0.5}\)>0.6>0.5,即a>c>b,故選\(\bbox[red,2pt]{(D)}\)。


解:
隨便找一個符合條件的數代入,例:令x=1,y=-1代入上式可得 4+5-3=6,故選\(\bbox[red,2pt]{(C)}\)。


解:
令\(\overline{B}=a且 \overline{HC}=b\),如上圖。
由於\(\overline{AD}=\overline{BF}=\overline{EH}=\overline{GC}\),
所以a+9=9+12+8=29,可得a=20;又9+12+8=8+b,可得b=21;
因此兩者相差21-20=1,故選\(\bbox[red,2pt]{(A)}\)。


解:
目前的數字1429,若要成為11的倍數,即(9+4)-(1+2)需為11的倍數,因此要將4變成5即可,故選\(\bbox[red,2pt]{(C)}\)。


解:
令\(\angle CAD=a\),如上圖。因此在\(\triangle ABC\)中,41+38+a+\(\angle 3\)=180,即\(\angle 3\)+a=101;
同理,\(\triangle ADE\)中,40+38+a+\(\angle 2\)=180,即\(\angle 2\)+a=102;
因此\(\angle 2-\angle 3=102-101=1,即\angle 2>\angle 3\),故選\(\bbox[red,2pt]{(B)}\)。


解:
事先預計a天完成訓練,則\(a\times 60=5\times 60+(a+3-5)\times 50\Rightarrow a=20\),共騎60a = 1200公里,故選\(\bbox[red,2pt]{(A)}\)。


解:
甲、乙、丙三個梯形的四內角均相等,但只有丙與丁的邊長成比例,故選\(\bbox[red,2pt]{(C)}\)。


解:
令甲容器底面半徑為a、乙容器底面半徑為b,則水的體積=$$12a^{ 2 }\pi =75b^{ 2 }\pi \Rightarrow a^2:b^2=75:12=25:4\Rightarrow a:b=5:2\\ \Rightarrow 周長比=2\pi a:2\pi b=5:2$$,故選\(\bbox[red,2pt]{(B)}\)。


解:
$$5x-24<2-x<2x+1\Rightarrow \begin{cases} 5x-24<2-x \\ 2-x<2x+1 \end{cases}\Rightarrow \begin{cases} 6x<26 \\ 1<3x \end{cases}\Rightarrow \frac { 1 }{ 3 } <x<\frac { 13 }{ 3 } \\ \Rightarrow b-a=\frac { 13 }{ 3 } -\frac { 1 }{ 3 } =\frac { 12 }{ 3 } =4$$,故選\(\bbox[red,2pt]{(A)}\)。


解:
令\(\angle BAC=a、\angle DAE=b及\angle ADC=c\),如上圖。則
\(\angle BCA=180-(134+a)=46-a、\angle ADE=180-(130+b)=50-b,及\angle ACD\) = 180-(47+c) = 133-c。
由於\(\overline{BC}平行\overline{ED}\Rightarrow \angle BCD+\angle EDC=180 \Rightarrow \) (46-a)+(133-c)+(50-b)+c=180 \(\Rightarrow\) a+b=49,故選\(\bbox[red,2pt]{(D)}\)。



解:
甲:乙=3:4及乙:丙=6:5\(\Rightarrow 甲:乙=9:12及乙:丙=12:10\Rightarrow 甲:乙:丙=9:12:10\),故選\(\bbox[red,2pt]{(C)}\)。


解:
由於(0,5)及(-2,5)為左右對稱,所以極值出現在x=(0-2)/2=-1。
x=-1代入直線,可求得y=3,即x=-1時有極值3,因此二次函數可以寫成\(y=a{(x+1)}^2+3\)。
函數通過(0, 5)可求得5=a+3,即a=2,函數為\(y=2{(x+1)}^2+3\);
(A)x=-4時,y=21;(B)x=-1時,y=3;(C)x=0時,y=5;(D)x=1時,y=11;
,故選\(\bbox[red,2pt]{(A)}\)。


解:
令公差=d,則總和S=\(a_1+a_2+\cdot +a_{500}=250(2a_1+499d)\)
又\(a_{91}+a_{410}>0\Rightarrow a_1+90d+a_1+409d>0\Rightarrow 2a_1+499d>0\)
$$\left( a_{ 110 }+a_{ 391 } \right) \left( a_{ 210 }+a_{ 291 } \right) =2\left( a_{ 191 }+a_{ 310 } \right) +24\\ \Rightarrow \left( a_{ 1 }+109d+a_{ 1 }+390d \right) \left( a_{ 1 }+209d+a_{ 1 }+290d \right) =2\left( a_{ 1 }+190d+a_{ 1 }+309d \right) +24\\ \Rightarrow \left( 2a_{ 1 }+499d \right) \left( 2a_{ 1 }+499d \right) =2\left( 2a_{ 1 }+499d \right) +24\\ \Rightarrow \left( 2a_{ 1 }+499d-6 \right) \left( 2a_{ 1 }+499d+4 \right) =0\\ \Rightarrow 2a_{ 1 }+499d=6(因為2a_1+499d>0,所以-4不合)\\ \Rightarrow 250\left( 2a_{ 1 }+499d \right) =1500\Rightarrow S=1500$$,故選\(\bbox[red,2pt]{(C)}\)。


解:
假設前15項有a個0、m個1及(15-a-m)個2,則後15項有(10-a)個0、(10-m)個1及(a+m-5)個2;
前15項的和=m+2(15-a-m)=16\(\Rightarrow\) m+2a=14
後15項的平方和=(10-m)+4(a+m-5)=26\(\Rightarrow\) 5m+4a=46
由上二式可求得a=3, m=8;b=10-a=7,a、b兩數相差7-3=4,故選\(\bbox[red,2pt]{(B)}\)。


解:

由於O為外心,所以O也是外接圓的圓心。圓心與各頂點形成的圓心角分別為 a, b, c, d, e (a+b+c+d+e=360),如上圖所示。$$\begin{cases} 2\angle B=a+b+c \\ 2\angle E=a+d+e \end{cases}\Rightarrow \begin{cases} a+b+c=240 \\ a+d+e=236 \end{cases}\Rightarrow a+\left( a+b+c+d+e \right) =476\\ \Rightarrow a=476-360=116$$,故選\(\bbox[red,2pt]{(A)}\)。



解:
很明顯可以看出L的方程式為: x+y=0
若C點坐標為(-40,0),則M的方程式為 x-y=0。但C點坐標為(-40,0)向左移一點,因此M也相對向左移一點,因此L與M的交點在第二象限,故選\(\bbox[red,2pt]{(B)}\)。


解:
在直角\(\triangle ONE\)中,\({\overline{OE}}^2=14^2+{\overline{NE}}^2\);
在直角\(\triangle OMA\)中,\({\overline{OE}}^2=15^2+{\overline{ME}}^2\);
因此\(14^2+{\overline{NE}}^2=15^2+{\overline{ME}}^2\Rightarrow \overline{NE}>\overline{ME}\)--------(1)

在直角\(\triangle ONC\)中,\(r^2=14^2+{\overline{NC}}^2\);
在直角\(\triangle OMA\)中,\(r^2=15^2+{\overline{MA}}^2\);
因此\(14^2+{\overline{NC}}^2=15^2+{\overline{MA}}^2\Rightarrow \overline{NC}>\overline{MA}\)--------(2)
(1)+(2)可得\(\overline{CE}>\overline{AE}\),故選\(\bbox[red,2pt]{(D)}\)。


解:
假設小智取出的球號為a、小慧取出的球號為b,則符合條件的事件為
(5,4)、(5,3)、(5,2)、(5,1)、(4,3)、(4,2)、(4,1)、(3,2)、(3,1)、(2,1)
共有10種情形,因機率為10/25=2/5,故選\(\bbox[red,2pt]{(C)}\)。


解:
令直線M與x軸交於D點、與y軸交於F點,過A點平行x軸與y軸交於E點,詳如上圖。
由A點坐標(-6,-4)可知:\(\overline{AE}=6及\overline{OE}=4\)。
在\(\triangle CAE中,\frac{\overline{CB}}{\overline{CA}}=\frac{\overline{BO}}{\overline{AE}} = \frac{\overline{CO}}{\overline{CE}}\Rightarrow \frac{2}{3}=\frac{\overline{BO}}{6}= \frac{\overline{CO}}{\overline{CO}+4}\Rightarrow \overline{BO}=4及\overline{CO}=8\)$$
\angle BAE+\angle EAF = 90 = \angle EFA+\angle EAF\Rightarrow \angle EFA=\angle BAE \Rightarrow \triangle CEA\sim\triangle AEF\\ \Rightarrow \frac{\overline{CE}}{\overline{AE}}=  \frac{\overline{AE}}{\overline{EF}}\Rightarrow \frac{12}{6}=\frac{6}{\overline{EF}}\Rightarrow \overline{EF}=3\\ 又\triangle AOF\sim\triangle CEA (\because \angle EFA=\angle BAE)\\ \Rightarrow \frac{\overline{DO}}{\overline{OM}}=\frac{\overline{CE}}{\overline{EA}}\Rightarrow \frac{\overline{DO}}{4+3}=\frac{8+4}{6}\Rightarrow \overline{DO}=14\Rightarrow m=-14$$,故選\(\bbox[red,2pt]{(C)}\)。



解:
令\(S_n\)代表在第n步驟後有多少棋子,則\(S_n=n^2+{(n-1)}^2, n=1,2,....\)。
令\(a_n\)代表在第n圈的棋子數,則\(a_n=S_n-S_{n-1}, n=2, 3, ....\)。
黑棋子數=\(a_1+a_3+a_5+a_7\),白棋子數=\(a_2+a_4+a_6+a_8\)。
\(S_1=1, S_2=5, S_3=3^2+2^2=13, S_4=4^2+3^2=25, S_5=5^+4^2=41\)
\(S_6=6^2+5^2=61, S_7=7^2+6^2=85, S_8=8^2+7^2=113\)
黑棋子數=1+(13-5)+(41-25)+(85-61)=49;白棋子數=4+(25-13)+(61-41)+(113-85)=64;
白棋子數比黑棋子數多64-49=15,故選\(\bbox[red,2pt]{(A)}\)。


解:
正面為1的牌,其背面為9,所以將此牌翻面後,數值差為8。
31變成47,相差47-31=16=兩張1翻成兩張9的差值。
因此桌上的牌原來是二張10、一張9、二張1及5張0,數字和=20+9+2=31;
將二張1翻成二張9後,數字和=20+9+18=47,故選\(\bbox[red,2pt]{(B)}\)。


解:
作一直線\(\overline{AG},\overline{AG}//\overline{DC},且交\overline{EF}於H\),如上圖。
在\(\triangle ABG中,\frac{\overline{AE}}{\overline{AB}}=\frac{\overline{EH}}{\overline{BG}}\Rightarrow \frac{2}{5}=\frac{4m-120}{5m-120}\)
\(\Rightarrow m=36\Rightarrow \overline{BG}=60\Rightarrow \overline{BC}=60+120=180\),故選\(\bbox[red,2pt]{(A)}\)。


解:
$$Q^{ 2 }-P^{ 2 }=24\Rightarrow \left( Q+P \right) \left( Q-P \right) =24\\ \Rightarrow \begin{matrix} Q+P & 24 & 12 & 8 & 6 & 4 & 3 & 2 & 1 \\ Q-P & 1 & 2 & 3 & 4 & 6 & 8 & 12 & 24 \end{matrix}$$由於(P+Q)>(Q-P),且Q={(P+Q)+(Q-P)}/2 必須是整數,所以只有Q+P=12,Q-P=2符合條件,即Q=7, P=5,故選\(\bbox[red,2pt]{(D)}\)。


解:
經過十分鐘(600秒)後,三人的坐標分別為甲(2400,0)、乙(0,3600)、丙(-1800,0)。
甲乙距離為\(\sqrt{2400^2+3600^2}=100\sqrt{1872}\),甲需要\(100\sqrt{1872}\div 4 \div \)60 分
由於\(\left({\frac{20\times 4\times 60}{100}}\right)^2={48}^2=2304>1872\),所以甲趕得上。
丙乙距離為\(\sqrt{1800^2+3600^2}=100\sqrt{1620}\),丙需要\(100\sqrt{1620}\div 3 \div 60 \)分
由於\(\left({\frac{20\times 3\times 60}{100}}\right)^2={36}^2=1296<1620\),所以丙趕不上。
故選\(\bbox[red,2pt]{(D)}\)。



解:
(A) 新加入的同學,一個體重比中位數低、一個比中位數高,所以中位數不會改變。
(B)新的算術平均數=\(\frac{31\times 47+45+47}{31+2}=46.9\),平均數已改變,故選\(\bbox[red,2pt]{(B)}\)。


解:
由題意可知,F為重心,因此\(\overline{CF}=2\overline{FD}=4\sqrt{3}\)
在直角\(\triangle CFB中,\overline{FB}=\overline{CF}\div\sqrt{3}=4\Rightarrow \triangle BFC=8\sqrt{3}\)
由於F為重心,所以AEFD面積=\(\triangle BFC=8\sqrt{3}\),故選\(\bbox[red,2pt]{(C)}\)。




解:
由A、B兩點坐標可求得\(\overline{AB}\)的方程式為y=-5x+25,因此D點坐標可假設為(a, -5a+25)
令E點坐標為(b, 3),E在直線\(\overline{AB}\)上,所以b=22/5
\(\triangle BCE面積=(\frac{22}{5}-1)\times 2 \div 2=\frac{17}{5}\)
梯形CEAO面積=\(\frac{\frac{17}{5}+5}{2}\times 3 = \frac{63}{5}\)
OABC面積=\(\frac{17}{5}+\frac{63}{5}=16\)
OCBD面積=OABC\(-\triangle OAD=3\triangle OAD\Rightarrow 4=\triangle OAD\)
\(\Rightarrow 5\times(25-5a)\div 2 = 4\Rightarrow a=\frac{117}{25}\Rightarrow D=(\frac{117}{25}, \frac{8}{5})\)


解:
(1)
M為\(\overline{AB}\)的中點,由於\(\triangle OAB\)為等腰,所以\(\overline{OM}垂直\overline{AB}\);同理\(\overline{PM}垂直\overline{AB}\)。因此O、M、P在同一直線上,\(\overline{OP}交\overline{AB}於M\),即M=C,所以\(\overline{AB}垂直\overline{OP}\)
(2)

 在直角\(\triangle OAP中,\overline{OA}=5及\overline{OP}=13,可求得\overline{AP}=12\)
令\(\overline{AC}=a\),則13=\(\overline{OC}+\overline{CP}=\sqrt{5^2-a^2}+\sqrt{12^2-a^2}\Rightarrow a=\frac{60}{13}\Rightarrow \overline{AB}=\frac{120}{13}\)


- END -

沒有留言:

張貼留言